2017 AMC 8 Problems/Problem 12

Revision as of 15:34, 22 November 2017 by Practicingmath (talk | contribs) (Created page with "==Problem 12== The smallest positive integer greater than 1 that leaves a remainder of 1 when divided by 4, 5, and 6 lies between which of the following pairs of numbers? <ma...")
(diff) ← Older revision | Latest revision (diff) | Newer revision → (diff)

Problem 12

The smallest positive integer greater than 1 that leaves a remainder of 1 when divided by 4, 5, and 6 lies between which of the following pairs of numbers?

$\textbf{(A) }2\text{ and }19\qquad\textbf{(B) }20\text{ and }39\qquad\textbf{(C) }40\text{ and }59\qquad\textbf{(D) }60\text{ and }79\qquad\textbf{(E) }80\text{ and }124$

Solution

Finding the LCM, we find out it is 60. Now, 60+1=61, and that is in the range of D.

See Also

2017 AMC 8 (ProblemsAnswer KeyResources)
Preceded by
Problem 11
Followed by
13
1 2 3 4 5 6 7 8 9 10 11 12 13 14 15 16 17 18 19 20 21 22 23 24 25
All AJHSME/AMC 8 Problems and Solutions

The problems on this page are copyrighted by the Mathematical Association of America's American Mathematics Competitions. AMC logo.png